1 Introduction

Let uvw be relatively prime positive integers greater than one and assume that the exponential Diophantine equation

$$\begin{aligned} u^{x}+v^{y}=w^{z} \end{aligned}$$
(1.1)

in positive integers xyz has a solution \((x_0, y_0, z_0).\) Two famous conjectures related to uniqueness of this solution \((x_0, y_0, z_0)\) are due to Jeśmanowicz and Terai with some restriction on (1.1). In 1956, Jeśmanowicz conjectured that if uv and w are any Pythagorean triples, i.e., positive integers satisfying \(u^{2}+v^{2}=w^{2},\) then the solution \((x_0,y_0,z_0)=(2,2,2)\) is the unique solution of (1.1) [5]. Another similar conjecture is proposed by Terai which states that if uvwpqr are fixed positive integers satisfying \( u^{p}+v^{q}=w^{r}\) with \(u,v,w,p,q,r\ge 2\), then the Eq.  (1.1) has unique positive integer solution \((x_0,y_0,z_0)=(p,q,r)\) [19, 20]. Exceptional cases are listed explicitly in [24]. Although both conjectures are proved to be true in many special cases, see for example [1, 3, 4, 6, 8, 10,11,12,13, 18, 21,22,23, 25], they are still remain an unsolved problem yet. We refer to [9, 17] for a detailed information on these two conjectures. In this note we study the exponential Diophantine equation

$$\begin{aligned} (m^2+m+1)^x+m^y=(m+1)^z \end{aligned}$$
(1.2)

where \(m>1\) is a positive integer, and we prove the following.

Theorem 1.1

Let \(m>1\) be a positive integer. If \( m > 3 \) then the Eq. (1.2) has only the positive integer solution \((x,y,z) =(1,1,2).\) For \( m=2 \) and \( m=3 \) the Eq. (1.1) has exactly two solutions, namely \((x,y,z) =(1,1,2), (2,5,4)\) and \((x,y,z) =(1,1,2), (1,5,4), \) respectively.

In the above theorem, we exclude the case \( m=1 \) just for preserving the exponent in the expression \( m^y. \) In fact, it is easy to see that the equation \( 3^x+1=2^z \) has only the positive integer solution \( (x, z)=(1, 2) \) by considering it modulo 8. For the next two values of m,  the Eq. (1.2) turns into the equations \( 7^x+2^y=3^z \) and \( 13^x+3^y=4^z, \) for which both of them have more than one solution [14, 26]. So the aim of this study is to give an answer to the question whether or not the Eq. (1.2) has any positive integer solutions other than \((x,y,z) =(1,1,2)\) when \( m>3.\) The proof depends on elementary congruence considerations and some results on linear forms in two \( m-\)adic logarithms.

2 Proof of Theorem 1.1

Lemma 2.1

Let (xyz) be a positive integer solution of the Eq. (1.2). The following conditions hold.

  1. 1.

    y is odd.

  2. 2.

    There exists an integer t such that \( |x-y |=(m+1)t. \)

Proof

  1. 1.

    By reducing Eq. (1.2) modulo \( m+1 \) we get that \( 1+(-1)^y \equiv 0 \pmod {(m+1)} \) which implies that y is odd since \( m>1.\)

  2. 2.

    If \(x=y\) then we may take \(t=0.\) So assume that \(|x-y |\ge 1.\) It is clear from (1.2) that \( z \ge 2. \) So we have that

    $$\begin{aligned} (m^2+m+1)^x +m^y&\equiv 0 \pmod {(m+1)^2} \\ (-m)^x +m^y&\equiv 0 \pmod {(m+1)^2} \\ m^{|x-y |} + (-1)^x&\equiv 0 \pmod {(m+1)^2} \\ ((m+1)-1)^{|x-y |} + (-1)^x&\equiv 0 \pmod {(m+1)^2} \\ (-1)^{|x-y |}+(-1)^{|x-y |-1 }(m+1) |x-y |+ (-1)^x&\equiv 0 \pmod {(m+1)^2} . \end{aligned}$$

    Taking into account that y is odd we get more precisely

    $$\begin{aligned} |x-y |&\equiv 0 \pmod {(m+1)}, \\ \end{aligned}$$

    which means that \( |x-y |=(m+1)t \) for some positive integer t.

\(\square \)

Lemma 2.2

If \( m \equiv 1 \pmod 4 \) then \( (x, y, z)=(1, 1,2) \) is the only solution of (1.2).

Proof

If \( z \le 2 \) then clearly \( (x, y, z)=(1, 1,2) \) is the only solution of (1.2). Assume that \( z \ge 3 \) and \( m=4k+1 \) for some positive integer k. If x is even, then \( (m^2+m+1)^x \equiv 1 \pmod 8 \) and \( m^{y-1} \equiv 1 \), so from (1.2) we get that \( 1+m \equiv 0 \pmod 8 \) which implies \( 2k+1 \equiv 0 \pmod 4, \) a contradiction. So x must be odd and hence \( (m^2+m+1)^x \equiv m^2+m+1 \pmod 8 .\) Then again considering (1.2) modulo 8 we get that

$$\begin{aligned} (m+1)^2&\equiv 0 \pmod 8 \\ (4k+2)^2&\equiv 0 \pmod 8 \\ 4&\equiv 0 \pmod 8 \end{aligned}$$

which is a contradiction. Hence, \( (x, y, z)=(1, 1,2) \) is the only solution of (1.2) when \( m \equiv 1 \pmod 4. \) \(\square \)

Lemma 2.3

Let (xyz) be a positive integer solution of the Eq. (1.2). Then x and y are relatively prime integers. In particular, \( x \ne y \) for \( z >2. \)

Proof

If \( z \le 2 \) then \( x=y=1 \) and hence the result is clear. So assume that \( z \ge 3 \) and that there exists an odd prime p such that \( x=x_{1}p \) and \( y=y_{1}p \) for some positive integers \( x_{1} \) and \( y_{1}\) since y is odd by Lemma 2.1. Let

$$\begin{aligned} K=(m^2+m+1)^{x_{1}}+m^{y_{1}}, \quad L= \frac{ (m^2+m+1)^{x_{1}p}+m^{y_{1}p} }{(m^2+m+1)^{x_{1}}+m^{y_{1}} } . \end{aligned}$$

So Eq. (1.2) is of the form

$$\begin{aligned} KL=(m+1)^z \end{aligned}$$
(2.1)

where \( \gcd (K,L)=1 \) or p. Note that \( K \equiv 0 \pmod {m+1}.\) Hence if \( \gcd (K,L)=1 \) then \( L=1 \) which is clearly impossible for \( p > 1.\) Thus, \( \gcd (K,L)=p.\) Let \( m+1=p^kq \) for some positive integer k such that \( \gcd (p,q)=1. \) From (2.1) we have that either \( K=p^{kz-1}q^z,\) \( L=p\) or \( K=pq^z,\) \( L=p^{kz-1}.\) For \( p>1 \) it is easy to see that

$$\begin{aligned} p(m^2+m+1)^{x_{1}} < (m^2+m+1)^{x_{1}p} \end{aligned}$$

and

$$\begin{aligned} pm^{y_{1}} < m^{y_{1}p} . \end{aligned}$$

So the case \(L=p\) leads to a contradiction. On the other hand it is known that \( p^2 \not \mid L ,\) see, for example, [15, P1.2], thus for the case \( K=pq^z,\) \( L=p^{kz-1}\) we have the only possibility \( kz-1=1 \) which is also a contradiction since \( z \ge 3.\) So, there do not exist such a prime p and hence x and y are relatively prime integers. \(\square \)

Let \( m' > 1 \) be an integer and let \( m'=p_1^{t_1} \ldots p_k^{t_k} \) be the prime factorization of \( m' \) for distinct primes \( p_i. \) The proof of Theorem 1.1 mainly depends on a result due to Bugeaud [2] on linear forms in two \( m'-\)adic logarithms. Let \( x_1 / y_1 \) and \( x_2 / y_2 \) be two non-zero rational numbers with \( x_1 / y_1 \ne \pm 1.\) In [2] Bugeaud provide an upper bound for the \(m'-\)adic valuation of

$$\begin{aligned} \Lambda = \left( x_1 / y_1 \right) ^{b_1} - \left( x_2 / y_2 \right) ^{b_2} \end{aligned}$$

whenever \(v_{p_i} \left( x_1 / y_1 \right) = v_{p_i} \left( x_2 / y_2 \right) =0 \) for all \( 1 \le i \le k \) where \( b_1 \) and \( b_2 \) are positive integers. Suppose that there exists a positive integer g which is coprime with \( p_1, \ldots , p_k \) such that for all prime \(p_i,\)

$$\begin{aligned} v_{p_i} \left( \left( \dfrac{x_1}{y_1} \right) ^g -1 \right) \ge {t_i} , v_{p_i} \left( \left( \dfrac{x_2}{y_2} \right) ^g -1 \right) \ge 1, 1 \le i \le k \end{aligned}$$
(2.2)

and

$$\begin{aligned} v_2 \left( \left( \dfrac{x_1}{y_1} \right) ^g -1 \right) \ge 2, v_2 \left( \left( \dfrac{x_2}{y_2} \right) ^g -1 \right) \ge 2 \text { if } 2\mid m'. \end{aligned}$$
(2.3)

Theorem 2.4

([2], Theorem 2]) Let \(A_{1}>1, A_{2}>1\) be real numbers such that

$$\begin{aligned} \log A_{i} \ge \max \{ \log |x_{i} |, \log |y_{i} |, \log m' \}, \quad i=1,2 \end{aligned}$$

and put

$$\begin{aligned} b'= \frac{b_{1}}{\log A_{2}}+\frac{b_{2}}{\log A_{1}}. \end{aligned}$$

Under the hypotheses (2.2) and (2.3) assume that \( x_1 / y_1 \) and \( x_2 / y_2 \) are multiplicatively independent. If \( m', \) \( b_1 \) and \( b_2 \) are relatively prime then we have the upper estimate

$$\begin{aligned} v_{m'} (\Lambda ) \le \dfrac{53.6g}{ \left( \log m' \right) ^4 } \left( \max \{ \log b' +\log \log m' +0.64 , 4 \log m' \} \right) ^2 \log A_{1} \log A_{2}. \end{aligned}$$

Now we apply the above theorem to the Eq. (1.2) by considering the \( (m+1)-\)adic valuation.

Lemma 2.5

Let \( m>7 \). If \( m \equiv 3 \pmod 4 \) or \( 2 \mid m \) then the Eq. (1.2) has only the positive integer solution \( (x, y, z)=(1, 1,2).\)

Proof

If \( z \le 2 \) then then the assertion is trivially true. Assume that \( z \ge 3 .\) Since y is odd, we rewrite the Eq. (1.2) as

$$\begin{aligned} (m+1)^z = (m^2+m+1)^x - (-m)^y \end{aligned}$$

and consider the \( (m+1)-\)adic valuation of \( (m^2+m+1)^x - (-m)^y.\) Since \( {(m+1)} \mid {m^2+m} \) , \( {(m+1)} \mid {-m-1}\), and also \(4 \mid {m^2+m} ,\) \( 4 \mid {(-m-1)} \) if \( m+1 \) is even. So, by Lemma 2.3, the hypotheses of Theorem 2.4 are satisfied for \( g=1 \) by taking \( x_{1}:=m^2+m+1 \) and \( x_{2}:=-m.\) Thus, from Theorem 2.4 we have the estimate

$$\begin{aligned} z\le & {} \dfrac{53.6}{ \left( \log {(m+1)} \right) ^4 } \left( \max \{ \log b' +\log \log {(m+1)} +0.64 , 4 \log {(m+1)} \} \right) ^2 \nonumber \\&\times \log {(m^2+m+1)} \log {m} \end{aligned}$$
(2.4)

where \( b'=\dfrac{x}{\log m} + \dfrac{y}{\log m^2+m+1}.\)

First assume that \( \log b' +\log \log {(m+1)} +0.64 > 4 \log {(m+1)}.\) We will show that this is not possible. Put \( M=\max \{x, y\}.\) Then

$$\begin{aligned} M \left( \dfrac{1}{\log m} +\dfrac{1}{\log (m^2+m+1)} \right) \ge b'> \dfrac{(m+1)^4}{ e^{0.64} \log {(m+1)}} \end{aligned}$$
(2.5)

and it follows that \( M > 2205\) since \( m \ge 8.\) On the other hand, from the Eq. (1.2) we see that

$$\begin{aligned} x \dfrac{\log (m^2+m+1)}{\log (m+1)}< z \quad \text {and} \quad y \dfrac{\log m}{\log (m+1)} < z. \end{aligned}$$

Thus \( M \dfrac{\log m}{\log (m+1)} < z.\) Combining this inequality and (2.4) together with (2.5) we get that

$$\begin{aligned} M \le 53.6 \left( \log M +U(m) +0.64 \right) ^2 V(m). \end{aligned}$$
(2.6)

where

$$\begin{aligned} U(m)= \log \left( \dfrac{\log {(m+1)}}{\log m} +\dfrac{\log {(m+1)}}{\log (m^2+m+1)} \right) \end{aligned}$$

and

$$\begin{aligned} V(m)= \dfrac{\log {(m^2+m+1)} }{ \left( \log {(m+1)} \right) ^3 }. \end{aligned}$$

For \( m \ge 8 ,\) both U(m) and V(m) are decreasing and \( U(m)\le U(8)<0.46 ,\) \( V(m) \le V(8)< 0.41 .\) Thus from (2.6) we get

$$\begin{aligned} M < 22 \left( \log M +1.1 \right) ^2 , \end{aligned}$$

which implies \( M<1576,\) a contradiction. So \( \log b' +\log \log {(m+1)} +0.64 \le 4 \log {(m+1)}.\) In this case from (2.4) we have that

$$\begin{aligned} z < 53.6 \times 16 \times W(m), \end{aligned}$$

where \( W(m)= \dfrac{\log m \log {(m^2+m+1)}}{( \log {(m+1)})^2}.\) In the above one can see that \( W(m)< 2 \) for all positive m, and hence we get that \( z<1716. \) Therefore, y is also bounded as \( 0.95 y<y\dfrac{\log m}{\log (m+1)}<z < 1716\) and hence \( y < 1807 \) for \( m \ge 8.\) Similarly, from the inequality

$$\begin{aligned} 1.96x< x\dfrac{\log m^2+m+1}{\log (m+1)}<z<1716 , \end{aligned}$$

we get \( x<876 \) for \( m \ge 8.\) Thus all xy and z are bounded. Moreover, from Lemma 2.1m is also bounded with \( m+1<M<1807.\) As a final step we checked with a short computer program in Maple that the equation (1.2) has no solution other than \( (x, y, z)=(1, 1,2)\) with these restrictions and those of Lemma 2.1 when m is in the range \( 8 \le m \le 1807.\) This completes the proof.

\(\square \)

Proof of Theorem 1.1

From Lemmas 2.2 and 2.5 it remains to check the Eq. (1.2) only for \( m \in \{2, 3, 4 , 6, 7 \} .\) The results for the equations \( 7^x+2^y=3^z,\) \( 13^x+3^y=4^z,\) and \( 57^x+7^y=8^z\) which corresponds to the case \( m=2, \) \( m=3 \) and \( m=7 \) in the Eq. (1.2) have already been established by a number of authors, at least [14, 26] and [16, Theorem 6] respectively. For \( m=4, \) the equation (1.2) turns into the equation \( 21^x+4^y=5^z. \) If x is even then by [7] this equation has no solution in positive integers whereas if x is odd then by [16, Lemma 6] it has only one solution, namely \( (x, y, z)=(1, 1,2).\) Finally we consider the equation \( 43^x+6^y=7^z \) for \( m=6.\) If x is odd then the equation has only one solution \( (x, y, z)=(1, 1,2)\) by [16, Lemma 6]. Suppose that x is even, say \( x=2X \). If \( y>1 \) then from the congruence \( 1 \equiv 7^z \pmod 8 \) we see that z is also even, say \( z=2Z. \) Thus we write

$$\begin{aligned} 2^y3^y=(7^Z-43^X)(7^Z+43^Z). \end{aligned}$$

Note that only one of the factors in the right hand side is divisible by 4 and \( 3 \not \mid 7^Z+43^Z.\) So we have two possibilities

$$\begin{aligned}&7^Z-43^X=2^{y-1}3^y \\&7^Z+43^Z=2 \end{aligned}$$

or

$$\begin{aligned}&7^Z-43^X=2 \cdot 3^y \\&7^Z+43^Z=2^{y-1} \end{aligned}$$

Clearly the first one is impossible. From the second one, we get that \( 7^Z=2^{y-2} +3^y. \) Reducing this equation modulo 3,  we find that y is even, a contradiction. Therefore, we conclude that \( y=1. \) Assume that x is even for otherwise the equation \( 43^x+6=7^z \) has only one solution \( (x, y, z)=(1, 1,2)\) from [16, Lemma 6]. Let \( x=2X .\) By reducing modulo 4 we see that z is odd, but it is easy to see that the equation \( 43^{2X}+6=7^{z} \) has no solution in positive integers when z is odd by considering it modulo 43. This completes the proof. \(\square \)